Mathematics
Mathematics, 16.12.2020 23:40, kiaramccurty

What is the solution to


What is the solution to

answer
Answers: 3

Other questions on the subject: Mathematics

image
Mathematics, 21.06.2019 15:30, bhhh7351
Which answer is the solution set to the inequality |x|< 9? a. x< -9 or x> 9 b. x> -9 and x< 9 c. x> -9 or x< 9 d. x< 9 and x< 9
Answers: 1
image
Mathematics, 21.06.2019 19:30, mary9590
Cone w has a radius of 8 cm and a height of 5 cm. square pyramid x has the same base area and height as cone w. paul and manuel disagree on how the volumes of cone w and square pyramid x are related. examine their arguments. which statement explains whose argument is correct and why? paul manuel the volume of square pyramid x is equal to the volume of cone w. this can be proven by finding the base area and volume of cone w, along with the volume of square pyramid x. the base area of cone w is π(r2) = π(82) = 200.96 cm2. the volume of cone w is one third(area of base)(h) = one third third(200.96)(5) = 334.93 cm3. the volume of square pyramid x is one third(area of base)(h) = one third(200.96)(5) = 334.93 cm3. the volume of square pyramid x is three times the volume of cone w. this can be proven by finding the base area and volume of cone w, along with the volume of square pyramid x. the base area of cone w is π(r2) = π(82) = 200.96 cm2. the volume of cone w is one third(area of base)(h) = one third(200.96)(5) = 334.93 cm3. the volume of square pyramid x is (area of base)(h) = (200.96)(5) = 1,004.8 cm3. paul's argument is correct; manuel used the incorrect formula to find the volume of square pyramid x. paul's argument is correct; manuel used the incorrect base area to find the volume of square pyramid x. manuel's argument is correct; paul used the incorrect formula to find the volume of square pyramid x. manuel's argument is correct; paul used the incorrect base area to find the volume of square pyramid x.
Answers: 3
image
Mathematics, 21.06.2019 23:30, teenybug56
If a runner who runs at a constant speed of p miles per hour runs a mile in exactly p minutes what is the integer closest to the value p
Answers: 2
image
Mathematics, 21.06.2019 23:30, mem8163
Graham’s monthly bank statement showed the following deposits and withdrawals. -$25.20, $42.75, -$22.04, -$18.50, $74.11. part a if grahams baldness in the account was $37.86 at the beginning of the month,! 27/4! 2// the account balance at the end of the month? ? plz i will give you 100 points
Answers: 1
Do you know the correct answer?
What is the solution to
...

Questions in other subjects:

Konu
Mathematics, 04.03.2020 20:20